IASbaba Prelims 60 Days Plan, Rapid Revision Series (RaRe)
Archives
Hello Friends
The 60 Days Rapid Revision (RaRe) Series is IASbaba’s Flagship Initiative recommended by Toppers and loved by the aspirants’ community every year.
It is the most comprehensive program which will help you complete the syllabus, revise and practice tests on a daily basis. The Programme on a daily basis includes
Daily Prelims MCQs from Static (Monday – Saturday)
- Daily Static Quiz will cover all the topics of static subjects – Polity, History, Geography, Economics, Environment and Science and technology.
- 20 questions will be posted daily and these questions are framed from the topics mentioned in the schedule.
- It will ensure timely and streamlined revision of your static subjects.
Daily Current Affairs MCQs (Monday – Saturday)
- Daily 5 Current Affairs questions, based on sources like ‘The Hindu’, ‘Indian Express’ and ‘PIB’, would be published from Monday to Saturday according to the schedule.
Daily CSAT Quiz (Monday – Friday)
- CSAT has been an Achilles heel for many aspirants.
- Daily 5 CSAT Questions will be published.
Note – Daily Test of 20 static questions, 10 current affairs, and 5 CSAT questions. (35 Prelims Questions) in QUIZ FORMAT will be updated on a daily basis.
To Know More about 60 Days Rapid Revision (RaRe) Series – CLICK HERE
60 Day Rapid Revision (RaRe) Series Schedule – CLICK HERE
Important Note
- Comment your Scores in the Comment Section. This will keep you accountable, responsible and sincere in days to come.
- It will help us come out with the Cut-Off on a Daily Basis.
- Let us know if you enjoyed today’s test 🙂
- You can post your comments in the given format
- (1) Your Score
- (2) Matrix Meter
- (3) New Learning from the Test
Test-summary
0 of 35 questions completed
Questions:
- 1
- 2
- 3
- 4
- 5
- 6
- 7
- 8
- 9
- 10
- 11
- 12
- 13
- 14
- 15
- 16
- 17
- 18
- 19
- 20
- 21
- 22
- 23
- 24
- 25
- 26
- 27
- 28
- 29
- 30
- 31
- 32
- 33
- 34
- 35
Information
The following Test is based on the syllabus of 60 Days Plan-2023 for UPSC IAS Prelims 2022.
To view Solutions, follow these instructions:
- Click on – ‘Start Test’ button
- Solve Questions
- Click on ‘Test Summary’ button
- Click on ‘Finish Test’ button
- Now click on ‘View Questions’ button – here you will see solutions and links.
You have already completed the test before. Hence you can not start it again.
Test is loading...
You must sign in or sign up to start the test.
You have to finish following test, to start this test:
Results
0 of 35 questions answered correctly
Your time:
Time has elapsed
You have scored 0 points out of 0 points, (0)
Average score |
|
Your score |
|
Categories
- Not categorized 0%
Pos. | Name | Entered on | Points | Result |
---|---|---|---|---|
Table is loading | ||||
No data available | ||||
- 1
- 2
- 3
- 4
- 5
- 6
- 7
- 8
- 9
- 10
- 11
- 12
- 13
- 14
- 15
- 16
- 17
- 18
- 19
- 20
- 21
- 22
- 23
- 24
- 25
- 26
- 27
- 28
- 29
- 30
- 31
- 32
- 33
- 34
- 35
- Answered
- Review
-
Question 1 of 35
1. Question
With reference to the Mission ‘Har Payment Digital’, consider the following statements:
- It was launched by Reserve Bank of India (RBI) during Digital Payments Awareness week to encourage digital payments adoption.
- As a part of the initiative, the RBI introduced the ’75 digital villages programme’ with the objective of converting 75 villages into digitally-enabled payment ecosystems.
Choose the correct code:
Correct
Solution (c)
Mission ‘Har Payment Digital’:
- The Reserve Bank of India (RBI) initiated this mission during the Digital Payments Awareness Week, which took place from 6th to 12th March 2023. (Hence statement 1 is correct)
- The primary objective of the mission is to encourage the widespread adoption of digital payments among the public.
- RBI’s call to action involves all stakeholders, including banks, payment system operators (PSOs), and digital payment users, to promote and adopt digital payment methods.
- Stakeholders are also encouraged to educate others about the benefits and advantages of using digital payment systems.
- As part of the initiative, RBI introduced the ’75 digital villages programme’ aimed at transforming 75 villages into digitally-enabled payment ecosystems. (Hence statement 2 is correct)
- Payment system operators (PSOs) will undertake the responsibility of adopting these villages and conducting awareness camps.
- The PSOs will also work to onboard merchants within these villages to accept digital payments, further promoting the use of cashless transactions.
Incorrect
Solution (c)
Mission ‘Har Payment Digital’:
- The Reserve Bank of India (RBI) initiated this mission during the Digital Payments Awareness Week, which took place from 6th to 12th March 2023. (Hence statement 1 is correct)
- The primary objective of the mission is to encourage the widespread adoption of digital payments among the public.
- RBI’s call to action involves all stakeholders, including banks, payment system operators (PSOs), and digital payment users, to promote and adopt digital payment methods.
- Stakeholders are also encouraged to educate others about the benefits and advantages of using digital payment systems.
- As part of the initiative, RBI introduced the ’75 digital villages programme’ aimed at transforming 75 villages into digitally-enabled payment ecosystems. (Hence statement 2 is correct)
- Payment system operators (PSOs) will undertake the responsibility of adopting these villages and conducting awareness camps.
- The PSOs will also work to onboard merchants within these villages to accept digital payments, further promoting the use of cashless transactions.
-
Question 2 of 35
2. Question
What are the major events in the early 1990s triggered the need for the New Industrial Policy of 1991 in India?
- The Gulf War
- The World Trade Organization’s formation
- The Great Depression
- India’s foreign exchange reserves had declined
Select the correct answer by using the code below:
Correct
Solution (c)
Background of New Industrial Policy of 1991
- The industrial policies of the past shaped the Indian economy’s nature and structure.
- By the early 1990s, there was a need to change the economy’s nature and structure.
- The Government of India (GoI) decided to change the industrial policy to bring about this change.
- The New Industrial Policy of 1991 was introduced to initiate economic reform in the country.
- This policy is considered more of a process than a mere policy due to its transformative nature.
- In June 1991, India faced a severe Balance of Payments (BoP) crisis due to various interconnected factors.
- The Gulf War (1990-91) resulted in higher oil prices, rapidly depleting India’s foreign reserves. (Hence 1 is correct)
- Private remittances from overseas Indian workers, especially from the Gulf region, sharply declined in the aftermath of the Gulf War.
- Inflation peaked at nearly 17 percent.
- The Central Government’s gross fiscal deficit reached 8.4 percent of the GDP.
- By June 1991, India’s foreign exchange reserves had declined to cover only two weeks of imports. (Hence 4 is correct)
- Initially, the focus was on macroeconomic stabilization, followed by reforms in industrial policy, trade, exchange rate policies, foreign investment policy, financial and tax systems, and public sector reforms.
- To address the BoP crisis, India received financial support from the International Monetary Fund (IMF) with certain conditions to be fulfilled.
- The IMF’s conditionality required structural re-adjustment in the Indian economy.
- Since past industrial policies shaped the economy’s structure, a new industrial policy was necessary to induce the desired changes.
- The New Industrial Policy of 1991 marked the beginning of broader economic reforms in India and aimed at fulfilling the IMF conditionality for structural readjustment.
Incorrect
Solution (c)
Background of New Industrial Policy of 1991
- The industrial policies of the past shaped the Indian economy’s nature and structure.
- By the early 1990s, there was a need to change the economy’s nature and structure.
- The Government of India (GoI) decided to change the industrial policy to bring about this change.
- The New Industrial Policy of 1991 was introduced to initiate economic reform in the country.
- This policy is considered more of a process than a mere policy due to its transformative nature.
- In June 1991, India faced a severe Balance of Payments (BoP) crisis due to various interconnected factors.
- The Gulf War (1990-91) resulted in higher oil prices, rapidly depleting India’s foreign reserves. (Hence 1 is correct)
- Private remittances from overseas Indian workers, especially from the Gulf region, sharply declined in the aftermath of the Gulf War.
- Inflation peaked at nearly 17 percent.
- The Central Government’s gross fiscal deficit reached 8.4 percent of the GDP.
- By June 1991, India’s foreign exchange reserves had declined to cover only two weeks of imports. (Hence 4 is correct)
- Initially, the focus was on macroeconomic stabilization, followed by reforms in industrial policy, trade, exchange rate policies, foreign investment policy, financial and tax systems, and public sector reforms.
- To address the BoP crisis, India received financial support from the International Monetary Fund (IMF) with certain conditions to be fulfilled.
- The IMF’s conditionality required structural re-adjustment in the Indian economy.
- Since past industrial policies shaped the economy’s structure, a new industrial policy was necessary to induce the desired changes.
- The New Industrial Policy of 1991 marked the beginning of broader economic reforms in India and aimed at fulfilling the IMF conditionality for structural readjustment.
-
Question 3 of 35
3. Question
Consider the following statements with respect to the incentive scheme to promote RuPay Debit Cards and low-value BHIM-UPI transactions:
- The scheme is being implemented by the Ministry of Finance, with a specified financial outlay.
- Financial incentives will be provided to banks to boost Point-of-Sale (PoS) and e-commerce transactions using RuPay Debit Cards and low-value BHIM-UPI transactions (P2M).
- The scheme aims to increase transparency in the government system, preventing leakages and fake recipients by directly transferring benefits to target beneficiaries through digital payment modes.
How many of the above statements are correct?
Correct
Solution (b)
The cabinet has recently approved an incentive scheme to promote RuPay Debit Cards and low-value BHIM-UPI transactions for the financial year 2022-23.
Features of the Scheme:
- The Ministry of Electronics and Information Technology is implementing the scheme, with a financial outlay of ₹2,600 crore. (Hence statement 1 is incorrect)
- Banks will receive financial incentives to encourage Point-of-Sale (PoS) and e-commerce transactions using RuPay Debit Cards and low-value BHIM-UPI transactions (P2M). (Hence statement 2 is correct)
- The scheme aims to promote UPI LITE and UPI 123PAY as economical and user-friendly digital payment solutions, increasing digital payment adoption across all sectors and population segments.
Significance of Digital Payments:
- Enhanced financial inclusion, offering easy access to accounts for receiving and making payments.
- Increased transparency in the government system, avoiding leakages and fake recipients by directly transferring benefits to target beneficiaries through digital payment modes.(Hence statement 3 is correct)
- Instant and convenient mode of payment, enabling quick and seamless transactions using BHIM-UPI and IMPS.
- Safe and secure digital payments with multiple layers of authentication.
- Enhanced credit access due to establishing a user’s financial footprint through digital payments, leading to increased access to formal financial services, including credit.
Challenges in Digital Payment Systems in India:
- India’s cash-dependent economy presents challenges in encouraging digital payments.
- Limited access to banks and cards in certain areas hinders the adoption of digital payment methods like UPI and mobile banking.
- Lack of financial literacy among a significant portion of the population.
- Risk of cyber fraud and privacy concerns affecting digital transactions.
- Cost and connectivity issues, as some users perceive cash as a free and more accessible payment method compared to digital payments.
Incorrect
Solution (b)
The cabinet has recently approved an incentive scheme to promote RuPay Debit Cards and low-value BHIM-UPI transactions for the financial year 2022-23.
Features of the Scheme:
- The Ministry of Electronics and Information Technology is implementing the scheme, with a financial outlay of ₹2,600 crore. (Hence statement 1 is incorrect)
- Banks will receive financial incentives to encourage Point-of-Sale (PoS) and e-commerce transactions using RuPay Debit Cards and low-value BHIM-UPI transactions (P2M). (Hence statement 2 is correct)
- The scheme aims to promote UPI LITE and UPI 123PAY as economical and user-friendly digital payment solutions, increasing digital payment adoption across all sectors and population segments.
Significance of Digital Payments:
- Enhanced financial inclusion, offering easy access to accounts for receiving and making payments.
- Increased transparency in the government system, avoiding leakages and fake recipients by directly transferring benefits to target beneficiaries through digital payment modes.(Hence statement 3 is correct)
- Instant and convenient mode of payment, enabling quick and seamless transactions using BHIM-UPI and IMPS.
- Safe and secure digital payments with multiple layers of authentication.
- Enhanced credit access due to establishing a user’s financial footprint through digital payments, leading to increased access to formal financial services, including credit.
Challenges in Digital Payment Systems in India:
- India’s cash-dependent economy presents challenges in encouraging digital payments.
- Limited access to banks and cards in certain areas hinders the adoption of digital payment methods like UPI and mobile banking.
- Lack of financial literacy among a significant portion of the population.
- Risk of cyber fraud and privacy concerns affecting digital transactions.
- Cost and connectivity issues, as some users perceive cash as a free and more accessible payment method compared to digital payments.
-
Question 4 of 35
4. Question
With reference to the Scheme for Promotion of Innovation, Rural Industries, and Entrepreneurship (ASPIRE), consider the following statements:
- The scheme aims to establish a network of technology and incubation centers to foster entrepreneurship, promote startups, and drive innovation in the agro-industry.
- Technical and research institutes, particularly those engaged in agro-based industries, are eligible to implement incubation and commercialization of business ideas under the scheme.
- The implementation of the scheme solely rests with the Ministry of Agriculture.
How many of the above statements are correct?
Correct
Solution (b)
Introduction to ASPIRE Scheme:
- The Scheme for Promotion of Innovation, Rural Industries, and Entrepreneurship (ASPIRE) aims to establish a network of technology centers and incubation centers to foster entrepreneurship, promote startups, and drive innovation in the agro-industry. (Hence statement 1 is correct)
- ASPIRE provides financial assistance for setting up Livelihood Business Incubators (LBI) or Technology Business Incubators (TBI).
Objectives of ASPIRE Scheme:
- Job Creation and Unemployment Reduction: The primary goal is to create new job opportunities and reduce unemployment in the country.
- Promote Entrepreneurship Culture: ASPIRE seeks to nurture a culture of entrepreneurship in India, encouraging individuals to start their own businesses.
- Grassroots Economic Development: The scheme aims at promoting economic development at the district level, focusing on the grassroots level.
- Addressing Unmet Social Needs: ASPIRE facilitates innovative business solutions to meet unmet social needs effectively.
- Strengthening MSME Sector Competitiveness: By promoting innovation, the scheme aims to enhance the competitiveness of Micro, Small, and Medium Enterprises (MSMEs).
Eligibility Criteria for ASPIRE:
- Technical/research institutes, especially those involved in agro-based industries, can implement incubation and commercialization of business ideas under the scheme. (Hence statement 2 is correct)
- These institutes will be designated as knowledge partners and will incubate new and existing technologies for commercialization.
- The scheme encourages the involvement of MSME/NSIC/KVIC/Coir Board/Other Ministries/Departments’ incubators and private incubators.
Application Process for ASPIRE:
- Interested parties can apply for the ASPIRE Scheme by submitting their applications to the ASPIRE Scheme Steering Committee of the Ministry of Micro, Small and Medium Enterprises (MSME). (Hence statement 3 is incorrect)
- The Scheme Steering Committee is responsible for overall policy, coordination, and management support.
Incorrect
Solution (b)
Introduction to ASPIRE Scheme:
- The Scheme for Promotion of Innovation, Rural Industries, and Entrepreneurship (ASPIRE) aims to establish a network of technology centers and incubation centers to foster entrepreneurship, promote startups, and drive innovation in the agro-industry. (Hence statement 1 is correct)
- ASPIRE provides financial assistance for setting up Livelihood Business Incubators (LBI) or Technology Business Incubators (TBI).
Objectives of ASPIRE Scheme:
- Job Creation and Unemployment Reduction: The primary goal is to create new job opportunities and reduce unemployment in the country.
- Promote Entrepreneurship Culture: ASPIRE seeks to nurture a culture of entrepreneurship in India, encouraging individuals to start their own businesses.
- Grassroots Economic Development: The scheme aims at promoting economic development at the district level, focusing on the grassroots level.
- Addressing Unmet Social Needs: ASPIRE facilitates innovative business solutions to meet unmet social needs effectively.
- Strengthening MSME Sector Competitiveness: By promoting innovation, the scheme aims to enhance the competitiveness of Micro, Small, and Medium Enterprises (MSMEs).
Eligibility Criteria for ASPIRE:
- Technical/research institutes, especially those involved in agro-based industries, can implement incubation and commercialization of business ideas under the scheme. (Hence statement 2 is correct)
- These institutes will be designated as knowledge partners and will incubate new and existing technologies for commercialization.
- The scheme encourages the involvement of MSME/NSIC/KVIC/Coir Board/Other Ministries/Departments’ incubators and private incubators.
Application Process for ASPIRE:
- Interested parties can apply for the ASPIRE Scheme by submitting their applications to the ASPIRE Scheme Steering Committee of the Ministry of Micro, Small and Medium Enterprises (MSME). (Hence statement 3 is incorrect)
- The Scheme Steering Committee is responsible for overall policy, coordination, and management support.
-
Question 5 of 35
5. Question
Consider the following statements with respect to the Production-Linked Incentive Scheme (PLI):
- The primary goals of the PLI scheme are to boost domestic manufacturing capabilities, promote import substitution, and generate employment opportunities.
- NITI Aayog has commenced the formulation of objective criteria for monitoring value addition by companies participating in the PLI schemes.
- The incentives are determined based on incremental sales, varying from 1% to 20%, depending on the specific sector.
How many of the above statements are correct?
Correct
Solution (c)
Production-Linked Incentive (PLI)
- NITI Aayog has initiated the development of objective criteria to track value addition by companies under Production-Linked Incentive (PLI) schemes. (Hence statement 2 is correct)
- The empowered group of secretaries, chaired by the Cabinet Secretary, is overseeing the implementation of PLI schemes, aiming for faster approvals, investments, and overall project turnaround.
- NITI Aayog aims to create a centralized database to monitor PLI scheme progress across sectors.
- An external agency (IFCI Ltd or SIDBI) will design and prepare the database to capture value addition, actual exports, and job creation.
- A state-level dashboard will be established to identify hurdles in the implementation.
Challenges Facing the PLI Scheme:
- Lack of Common Parameters: Currently, different ministries monitor value addition in their respective PLI schemes, leading to a lack of standardized comparison.
- Steep Targets: Some companies find it difficult to meet the steep targets required to qualify for incentives under the PLI scheme.
- Reliance on Limited Supply Chains: Domestic companies relying on one or two supply chains face challenges in qualifying for the incentive due to supply disruptions.
About the PLI Scheme:
- Launched in March 2020, the PLI scheme aims to enhance domestic manufacturing capabilities, increase import substitution, and generate employment.(Hence statement 1 is correct)
- The scheme covers various sectors, including mobile and allied component manufacturing, electrical component manufacturing, medical devices, and others.
- Incentives are calculated based on incremental sales, ranging from 1% to 20%, depending on the sector.(Hence statement 3 is correct)
- The PLI scheme is introduced to reduce India’s reliance on foreign countries, support labor-intensive sectors, reduce import bills, and promote domestic production. Foreign companies are encouraged to set up units in India, while domestic enterprises expand their production units.
Incorrect
Solution (c)
Production-Linked Incentive (PLI)
- NITI Aayog has initiated the development of objective criteria to track value addition by companies under Production-Linked Incentive (PLI) schemes. (Hence statement 2 is correct)
- The empowered group of secretaries, chaired by the Cabinet Secretary, is overseeing the implementation of PLI schemes, aiming for faster approvals, investments, and overall project turnaround.
- NITI Aayog aims to create a centralized database to monitor PLI scheme progress across sectors.
- An external agency (IFCI Ltd or SIDBI) will design and prepare the database to capture value addition, actual exports, and job creation.
- A state-level dashboard will be established to identify hurdles in the implementation.
Challenges Facing the PLI Scheme:
- Lack of Common Parameters: Currently, different ministries monitor value addition in their respective PLI schemes, leading to a lack of standardized comparison.
- Steep Targets: Some companies find it difficult to meet the steep targets required to qualify for incentives under the PLI scheme.
- Reliance on Limited Supply Chains: Domestic companies relying on one or two supply chains face challenges in qualifying for the incentive due to supply disruptions.
About the PLI Scheme:
- Launched in March 2020, the PLI scheme aims to enhance domestic manufacturing capabilities, increase import substitution, and generate employment.(Hence statement 1 is correct)
- The scheme covers various sectors, including mobile and allied component manufacturing, electrical component manufacturing, medical devices, and others.
- Incentives are calculated based on incremental sales, ranging from 1% to 20%, depending on the sector.(Hence statement 3 is correct)
- The PLI scheme is introduced to reduce India’s reliance on foreign countries, support labor-intensive sectors, reduce import bills, and promote domestic production. Foreign companies are encouraged to set up units in India, while domestic enterprises expand their production units.
-
Question 6 of 35
6. Question
With reference to the Prime Minister Street Vendor’s AtmaNirbharNidhi (PM-SVANidhi), consider the following statements:
- The PM-SVANidhi is a Central Sector Scheme by the Ministry of Housing and Urban Affairs, providing support to street vendors through working capital loans and promoting digital transactions.
- This scheme is exclusively available to street vendors operating in rural areas engaged in vending.
Choose the correct code:
Correct
Solution (a)
PM-SVANidhi Scheme:
- The Prime Minister Street Vendor’s AtmaNirbharNidhi (PM-SVANidhi) is a Central Sector Scheme launched by the Ministry of Housing and Urban Affairs. This scheme aims to support and empower street vendors by providing them with access to working capital loans and promoting digital transactions. (Hence statement 1 is correct)
- It also incentivizes timely loan repayments and rewards vendors for embracing digital payment methods. By promoting ease of doing business for street vendors, the scheme plays a crucial role in fostering entrepreneurship, financial inclusion, and overall economic growth.
- Over 46.54 lakh small working capital loans have been disbursed to street vendors under the Prime Minister Street Vendor’s AtmaNirbharNidhi (PM-SVANidhi) since its launch on June 1, 2020.
- Out of these loans, approximately 40% (18,50,987) have been repaid so far.
Objectives:
- Facilitate working capital loans for street vendors.
- Incentivize regular loan repayment.
- Reward digital transactions.
Three-Tier Loan Structure:
- 1st loan of up to ₹10,000
- 2nd loan of up to ₹20,000
- 3rd term loan of up to ₹50,000
Lending Agencies:
- Microfinance Institutions, Non-Banking Financial Companies, and Self-Help Groups can participate due to their proximity to urban poor and street vendors.
Eligibility:
- Available in States/Union Territories (UTs) that have notified Rules and Scheme under Street Vendors (Protection of Livelihood and Regulation of Street Vending) Act, 2014.
- Street vendors engaged in vending in urban areas are eligible. (Hence statement 2 is incorrect)
Incorrect
Solution (a)
PM-SVANidhi Scheme:
- The Prime Minister Street Vendor’s AtmaNirbharNidhi (PM-SVANidhi) is a Central Sector Scheme launched by the Ministry of Housing and Urban Affairs. This scheme aims to support and empower street vendors by providing them with access to working capital loans and promoting digital transactions. (Hence statement 1 is correct)
- It also incentivizes timely loan repayments and rewards vendors for embracing digital payment methods. By promoting ease of doing business for street vendors, the scheme plays a crucial role in fostering entrepreneurship, financial inclusion, and overall economic growth.
- Over 46.54 lakh small working capital loans have been disbursed to street vendors under the Prime Minister Street Vendor’s AtmaNirbharNidhi (PM-SVANidhi) since its launch on June 1, 2020.
- Out of these loans, approximately 40% (18,50,987) have been repaid so far.
Objectives:
- Facilitate working capital loans for street vendors.
- Incentivize regular loan repayment.
- Reward digital transactions.
Three-Tier Loan Structure:
- 1st loan of up to ₹10,000
- 2nd loan of up to ₹20,000
- 3rd term loan of up to ₹50,000
Lending Agencies:
- Microfinance Institutions, Non-Banking Financial Companies, and Self-Help Groups can participate due to their proximity to urban poor and street vendors.
Eligibility:
- Available in States/Union Territories (UTs) that have notified Rules and Scheme under Street Vendors (Protection of Livelihood and Regulation of Street Vending) Act, 2014.
- Street vendors engaged in vending in urban areas are eligible. (Hence statement 2 is incorrect)
-
Question 7 of 35
7. Question
Consider the following statements with respect to the South India’s first Industrial Corridor Project:
- Tumakuru, Karnataka, has been chosen as the location for the implementation of the project under the National Industrial Corridor programme (NICP).
- Notable aspects of NICP comprise the preparation of land parcels for prompt allocation to manufacturing units and securing necessary statutory clearances beforehand.
- The National Industrial Corridor Development & Implementation Trust (NICDIT) is responsible for unified development of the corridors.
How many of the above statements are correct?
Correct
Solution (c)
South India’s 1st Industrial Corridor Project:
- The project will be implemented in Tumakuru, Karnataka, as part of the National Industrial Corridor programme (NICP). (Hence statement 1 is correct)
- Krishnapatnam (Andhra Pradesh), Tumakuru, and Ponneri (Tamil Nadu) are priority nodes under the Chennai Bengaluru Industrial Corridor (CBIC) of NICP.
- NICP aims to promote world-class manufacturing facilities and futuristic industrial cities in India, providing plug and play infrastructure for large-scale manufacturing units.
- It includes the development of various industrial corridors like Delhi Mumbai Industrial Corridor (DMIC), Amritsar-Kolkata, CBIC, Vizag-Chennai, and East Coast.
- Industrial corridors are crucial to foster global manufacturing hubs, smart cities, and quality infrastructure ahead of demand.
- They aim to increase the contribution of manufacturing to GDP, generate employment, and improve living conditions for the workforce.
- Key features of NICP include developing land parcels for immediate allotment to manufacturing units and obtaining statutory clearances in advance.(Hence statement 2 is correct)
- The National Industrial Corridor Development & Implementation Trust (NICDIT) is responsible for unified development of the corridors.(Hence statement 3 is correct)
Incorrect
Solution (c)
South India’s 1st Industrial Corridor Project:
- The project will be implemented in Tumakuru, Karnataka, as part of the National Industrial Corridor programme (NICP). (Hence statement 1 is correct)
- Krishnapatnam (Andhra Pradesh), Tumakuru, and Ponneri (Tamil Nadu) are priority nodes under the Chennai Bengaluru Industrial Corridor (CBIC) of NICP.
- NICP aims to promote world-class manufacturing facilities and futuristic industrial cities in India, providing plug and play infrastructure for large-scale manufacturing units.
- It includes the development of various industrial corridors like Delhi Mumbai Industrial Corridor (DMIC), Amritsar-Kolkata, CBIC, Vizag-Chennai, and East Coast.
- Industrial corridors are crucial to foster global manufacturing hubs, smart cities, and quality infrastructure ahead of demand.
- They aim to increase the contribution of manufacturing to GDP, generate employment, and improve living conditions for the workforce.
- Key features of NICP include developing land parcels for immediate allotment to manufacturing units and obtaining statutory clearances in advance.(Hence statement 2 is correct)
- The National Industrial Corridor Development & Implementation Trust (NICDIT) is responsible for unified development of the corridors.(Hence statement 3 is correct)
-
Question 8 of 35
8. Question
Consider the following:
- The Kisan Credit Card (KCC) scheme was introduced in 2018 for issue of Kisan Credit Cards to farmers on the basis of their holdings for uniform adoption by the banks so that farmers may use them to readily purchase agriculture inputs such as seeds, fertilizers, pesticides etc.
- Kisan Rin Portal has been launched recently to revolutionize access to credit services under the Kisan Credit Card (KCC) scheme, particularly for farmers.
Choose the correct code:
Correct
Solution (c)
Recently, the Indian govt launched the Kisan Rin Portal with the weather portal WIND.
- Government launches Kisan Rin Portal under the Kisan Credit Card scheme to revolutionize the agriculture sector of India, along with the weather information Network Data Systems (WINDS) portal.
The WIND portal: –
- Launched: July 2023.
- Objective: to leverage advanced weather data analytics and to give stakeholders actionable insight to make informed weather decisions on agriculture.
- The portal also provides a comprehensive manual to stakeholders for an in-depth understanding of the portal functionalities, data interpretations, and effective utilization, empowering farmers, policymakers, and various agricultural entities to make well-informed decisions.
About Kisan Rin Portal: –
- Launched: September,2023.
- Objective: to revolutionize access to credit services under the Kisan Credit Card (KCC) scheme, particularly for farmers.
- The Kisan Credit Card (KCC) scheme is a credit scheme introduced in August 1998 by Indian public sector banks.
- Implementation: Ministry of Agriculture and Farmers’ Welfare (MoA&FW), Ministry of Finance, Reserve Bank of India (RBI), and National Bank for Agriculture and Rural Development (NABARD).
Salient Features: –
- The Kisan Rin portal serves as an integrated hub.
- It offers a comprehensive view of farmer data.
- It will also show the progress in scheme utilization.
- It will provide farmers easy access to subsidized loans under the Kisan Credit Card (KCC) scheme.
- The Kisan Rin digital platform will offer:-
- a comprehensive view of farmer data
- scheme utilization progress
- loan disbursement specifics
- interest subvention claims
- It will facilitate farmers in availing of subsidized agricultural credit through the Modified Interest Subvention Scheme (MISS).
Incorrect
Solution (c)
Recently, the Indian govt launched the Kisan Rin Portal with the weather portal WIND.
- Government launches Kisan Rin Portal under the Kisan Credit Card scheme to revolutionize the agriculture sector of India, along with the weather information Network Data Systems (WINDS) portal.
The WIND portal: –
- Launched: July 2023.
- Objective: to leverage advanced weather data analytics and to give stakeholders actionable insight to make informed weather decisions on agriculture.
- The portal also provides a comprehensive manual to stakeholders for an in-depth understanding of the portal functionalities, data interpretations, and effective utilization, empowering farmers, policymakers, and various agricultural entities to make well-informed decisions.
About Kisan Rin Portal: –
- Launched: September,2023.
- Objective: to revolutionize access to credit services under the Kisan Credit Card (KCC) scheme, particularly for farmers.
- The Kisan Credit Card (KCC) scheme is a credit scheme introduced in August 1998 by Indian public sector banks.
- Implementation: Ministry of Agriculture and Farmers’ Welfare (MoA&FW), Ministry of Finance, Reserve Bank of India (RBI), and National Bank for Agriculture and Rural Development (NABARD).
Salient Features: –
- The Kisan Rin portal serves as an integrated hub.
- It offers a comprehensive view of farmer data.
- It will also show the progress in scheme utilization.
- It will provide farmers easy access to subsidized loans under the Kisan Credit Card (KCC) scheme.
- The Kisan Rin digital platform will offer:-
- a comprehensive view of farmer data
- scheme utilization progress
- loan disbursement specifics
- interest subvention claims
- It will facilitate farmers in availing of subsidized agricultural credit through the Modified Interest Subvention Scheme (MISS).
-
Question 9 of 35
9. Question
How did the New Economic Policy impact India’s trade policies?
Correct
Solution (c)
The New Economic Policy (NEP):
The New Economic Policy (NEP) aimed to liberalize and open up the Indian economy to international markets, moving away from the earlier policy of import substitution and protectionism. As part of this liberalization process, the NEP focused on promoting export-oriented growth and reducing barriers to imports. (Hence option (c) is correct answer)
Here’s how it impacted India’s trade policies:
- Export Promotion: The NEP introduced various measures to encourage and support Indian exporters. It provided incentives, subsidies, and export-oriented infrastructure to boost exports. The goal was to make Indian products competitive in the global market and increase the country’s share in international trade.
- Reduced Import Barriers: The NEP aimed to dismantle trade barriers and reduce restrictions on imports. It gradually lowered tariffs and import duties, making it easier for foreign goods to enter the Indian market. This move was intended to increase access to foreign technology, capital goods, and consumer products, fostering competition and efficiency.
- Foreign Investment: The NEP allowed greater foreign direct investment (FDI) in various sectors of the economy. This not only brought in much-needed capital but also helped in technology transfer, improved management practices, and access to global markets.
- Exchange Rate Liberalization: The NEP moved towards a more market-determined exchange rate system, reducing government control over currency valuation. A more flexible exchange rate allowed the Indian rupee to find its equilibrium value based on market forces, promoting export competitiveness.
- Export Processing Zones (EPZs): The NEP established Export Processing Zones (EPZs), later renamed Special Economic Zones (SEZs), to facilitate export-oriented industries with favorable regulatory and tax conditions.
Incorrect
Solution (c)
The New Economic Policy (NEP):
The New Economic Policy (NEP) aimed to liberalize and open up the Indian economy to international markets, moving away from the earlier policy of import substitution and protectionism. As part of this liberalization process, the NEP focused on promoting export-oriented growth and reducing barriers to imports. (Hence option (c) is correct answer)
Here’s how it impacted India’s trade policies:
- Export Promotion: The NEP introduced various measures to encourage and support Indian exporters. It provided incentives, subsidies, and export-oriented infrastructure to boost exports. The goal was to make Indian products competitive in the global market and increase the country’s share in international trade.
- Reduced Import Barriers: The NEP aimed to dismantle trade barriers and reduce restrictions on imports. It gradually lowered tariffs and import duties, making it easier for foreign goods to enter the Indian market. This move was intended to increase access to foreign technology, capital goods, and consumer products, fostering competition and efficiency.
- Foreign Investment: The NEP allowed greater foreign direct investment (FDI) in various sectors of the economy. This not only brought in much-needed capital but also helped in technology transfer, improved management practices, and access to global markets.
- Exchange Rate Liberalization: The NEP moved towards a more market-determined exchange rate system, reducing government control over currency valuation. A more flexible exchange rate allowed the Indian rupee to find its equilibrium value based on market forces, promoting export competitiveness.
- Export Processing Zones (EPZs): The NEP established Export Processing Zones (EPZs), later renamed Special Economic Zones (SEZs), to facilitate export-oriented industries with favorable regulatory and tax conditions.
-
Question 10 of 35
10. Question
Consider the following statements with respect to the National Monetisation Pipeline (NMP):
- NITI Aayog developed the NMP in consultation with infrastructure line ministries to unlock value in brownfield projects through private sector engagement.
- Under this scheme the private sector is granted revenue rights, while the ownership of projects remains with the government.
Choose the correct code:
Correct
Solution (c)
National Monetisation Pipeline (NMP):
Development and Mandate:
- NITI Aayog developed the NMP in consultation with infrastructure line ministries.
- Mandated under Union Budget 2021-22 for ‘Asset Monetisation’ to unlock value in brownfield projects through private sector engagement.
Objective:
- Aims to unlock value in brownfield projects by engaging the private sector. (Hence statement 1 is correct)
- Revenue rights transferred to private sector, but ownership in projects remains with the government. (Hence statement 2 is correct)
- Funds generated used for infrastructure development across the country.
Incorrect
Solution (c)
National Monetisation Pipeline (NMP):
Development and Mandate:
- NITI Aayog developed the NMP in consultation with infrastructure line ministries.
- Mandated under Union Budget 2021-22 for ‘Asset Monetisation’ to unlock value in brownfield projects through private sector engagement.
Objective:
- Aims to unlock value in brownfield projects by engaging the private sector. (Hence statement 1 is correct)
- Revenue rights transferred to private sector, but ownership in projects remains with the government. (Hence statement 2 is correct)
- Funds generated used for infrastructure development across the country.
-
Question 11 of 35
11. Question
What does the term “ANTARDRISHTI”, which has been recently featured in the news, pertain to?
Correct
Solution (c)
ANTARDRISHTI, a Financial Inclusion Dashboard introduced by the RBI, has been recently launched. This dashboard offers the necessary tools to evaluate and track the advancement of financial inclusion by capturing pertinent indicators. Moreover, this platform facilitates the measurement of the degree of financial exclusion in intricate geographical regions nationwide, thus enabling targeted solutions for such areas. (Hence option (c) is correct)
Incorrect
Solution (c)
ANTARDRISHTI, a Financial Inclusion Dashboard introduced by the RBI, has been recently launched. This dashboard offers the necessary tools to evaluate and track the advancement of financial inclusion by capturing pertinent indicators. Moreover, this platform facilitates the measurement of the degree of financial exclusion in intricate geographical regions nationwide, thus enabling targeted solutions for such areas. (Hence option (c) is correct)
-
Question 12 of 35
12. Question
Soil Health Cards (SHC) serves as printed report cards issued to farmers at three-year intervals, indicating their soil’s condition. Which of the following outcomes can be expected from Soil Health Cards?
- Heightened financial pressure on the government
- Balanced utilisation of fertilisers
- Improved land management
- Reduction in the import bill
Select the correct answer by using the code below:
Correct
Solution (c)
- A Soil Health Card (SHC) is an issued report card provided to farmers once every three years, conveying the state of their soil across 12 parameters. Alongside, it includes recommendations for utilisingfertilisers and other soil enhancements effectively.
- The current consumption pattern of Nitrogen – Phosphorus – Potassium (NPK) in the country is significantly skewed towards an imbalanced ratio of 4:2:1. SHC serves to offer comprehensive field-specific soil fertility reports, facilitating the appropriate distribution of fertilisers.
- The promotion of an integrated nutrient system is anticipated to curtail chemical fertiliser usage by 20%, thus alleviating the fiscal burden on the Government. This step is essential, as the fertiliser sector contributes significantly to the total subsidies and power consumption within the nation. (Hence statement 1 is incorrect)
- India relies on substantial fertiliser imports to meet its requirements, such as importing around 25-30% of the Urea demand. Therefore, employing fertiliser usage based on soil tests will lead to reduced import costs, fostering enhanced yields per unit of cultivation.
- With the passage of time, Soil Health Cards (SHC) can ascertain alterations in soil health stemming from land management practices.
Incorrect
Solution (c)
- A Soil Health Card (SHC) is an issued report card provided to farmers once every three years, conveying the state of their soil across 12 parameters. Alongside, it includes recommendations for utilisingfertilisers and other soil enhancements effectively.
- The current consumption pattern of Nitrogen – Phosphorus – Potassium (NPK) in the country is significantly skewed towards an imbalanced ratio of 4:2:1. SHC serves to offer comprehensive field-specific soil fertility reports, facilitating the appropriate distribution of fertilisers.
- The promotion of an integrated nutrient system is anticipated to curtail chemical fertiliser usage by 20%, thus alleviating the fiscal burden on the Government. This step is essential, as the fertiliser sector contributes significantly to the total subsidies and power consumption within the nation. (Hence statement 1 is incorrect)
- India relies on substantial fertiliser imports to meet its requirements, such as importing around 25-30% of the Urea demand. Therefore, employing fertiliser usage based on soil tests will lead to reduced import costs, fostering enhanced yields per unit of cultivation.
- With the passage of time, Soil Health Cards (SHC) can ascertain alterations in soil health stemming from land management practices.
-
Question 13 of 35
13. Question
The ‘Consumer Confidence Survey’ in India is conducted by which of the following:
Correct
Solution (a)
Since June 2010, the Reserve Bank has been executing the Consumer Confidence Survey (CCS). This survey gathers subjective insights regarding economic conditions, household situations, income, expenditures, pricing, and employment opportunities. The survey’s findings are derived from respondents’ perspectives and may not necessarily align with those of the Reserve Bank of India. (Hence option (a) is correct)
Incorrect
Solution (a)
Since June 2010, the Reserve Bank has been executing the Consumer Confidence Survey (CCS). This survey gathers subjective insights regarding economic conditions, household situations, income, expenditures, pricing, and employment opportunities. The survey’s findings are derived from respondents’ perspectives and may not necessarily align with those of the Reserve Bank of India. (Hence option (a) is correct)
-
Question 14 of 35
14. Question
Consider the following statements concerning Goods and Services Tax (GST):
- It has resulted in tax harmonisation.
- No GST is imposed throughout the supply chain for exports.
Choose the accurate response using the provided code:
Correct
Solution (c)
- The implementation of GST has resulted in a decreased count of taxes, establishing a uniform tax rate that is applicable nationwide to all goods and services. This standardisation of taxes is referred to as tax harmonisation. In the case of exports, the government refunds the GST paid to suppliers, effectively eliminating GST across the entire supply chain. This practice is known as ‘zero-rated’ exports. (Hence both the statements are correct)
- During the introduction of the Goods and Services Tax (GST) by the Central government, concerns arose among states regarding potential drops in tax revenue and limitations on their ability to impose extra taxes within the GST framework. To address these worries, the Indian government calculated the annual growth of state indirect taxes for three consecutive years—2012-13 to 2013-14, 2013-14 to 2014-15, and 2014-15 to 2015-16—and determined an average annual growth rate of 14%. The government assured states that if their indirect revenue growth fell below 14% annually post-GST implementation, a Cess would be imposed on luxury and demerit goods. The revenue generated would then be provided to states as compensation over a five-year period from July 1, 2017, to June 30, 2022. To facilitate this, the Goods and Services Tax (Compensation to States) Act 2017 was enacted.
- Under this framework, the Indian government collects the GST compensation cess, placing it in the Consolidated Fund of India before transferring it to the “GST Compensation Fund” within the Public Account of India. This is because the money is earmarked for states and not considered the government’s funds. Eventually, the funds are transferred to the respective states and union territories.
- Following the initiation of GST, revenue shortfalls arose and states received compensation. However, the COVID-19 pandemic exacerbated these shortfalls to an extent where increasing cess on luxury and demerit goods would not suffice to cover the significant deficit. As of November 2020, a dispute emerged regarding how to address this situation. Ultimately, a resolution was reached that the “Centre” (Central government) would borrow the additional shortfall amount caused by the pandemic. This borrowed amount would then be provided to states in the form of a loan on a “back-to-back” basis. States would be responsible for repaying the principal and interest on this loan in the future. Notably, the borrowing cost for the Centre is lower compared to that for states due to the Centre’s inability to default and its stronger credit rating.
Incorrect
Solution (c)
- The implementation of GST has resulted in a decreased count of taxes, establishing a uniform tax rate that is applicable nationwide to all goods and services. This standardisation of taxes is referred to as tax harmonisation. In the case of exports, the government refunds the GST paid to suppliers, effectively eliminating GST across the entire supply chain. This practice is known as ‘zero-rated’ exports. (Hence both the statements are correct)
- During the introduction of the Goods and Services Tax (GST) by the Central government, concerns arose among states regarding potential drops in tax revenue and limitations on their ability to impose extra taxes within the GST framework. To address these worries, the Indian government calculated the annual growth of state indirect taxes for three consecutive years—2012-13 to 2013-14, 2013-14 to 2014-15, and 2014-15 to 2015-16—and determined an average annual growth rate of 14%. The government assured states that if their indirect revenue growth fell below 14% annually post-GST implementation, a Cess would be imposed on luxury and demerit goods. The revenue generated would then be provided to states as compensation over a five-year period from July 1, 2017, to June 30, 2022. To facilitate this, the Goods and Services Tax (Compensation to States) Act 2017 was enacted.
- Under this framework, the Indian government collects the GST compensation cess, placing it in the Consolidated Fund of India before transferring it to the “GST Compensation Fund” within the Public Account of India. This is because the money is earmarked for states and not considered the government’s funds. Eventually, the funds are transferred to the respective states and union territories.
- Following the initiation of GST, revenue shortfalls arose and states received compensation. However, the COVID-19 pandemic exacerbated these shortfalls to an extent where increasing cess on luxury and demerit goods would not suffice to cover the significant deficit. As of November 2020, a dispute emerged regarding how to address this situation. Ultimately, a resolution was reached that the “Centre” (Central government) would borrow the additional shortfall amount caused by the pandemic. This borrowed amount would then be provided to states in the form of a loan on a “back-to-back” basis. States would be responsible for repaying the principal and interest on this loan in the future. Notably, the borrowing cost for the Centre is lower compared to that for states due to the Centre’s inability to default and its stronger credit rating.
-
Question 15 of 35
15. Question
With reference to the Public Distribution System (PDS), consider the following statements:
- PDS aims to ensure equitable distribution, stabilize prices, and support poverty-alleviation programs.
- The Department of Agriculture and Rural Development is tasked with overseeing the procurement and storage of food grains.
- It serves as a preventive measure against stockpiling and illicit market activities.
How many of the above statements are correct?
Correct
Solution (b)
Public Distribution System (PDS): Overview, Functioning, Limitations, and Revamping
Overview and Functioning of PDS:
- PDS plays a vital role in ensuring food security by supplying subsidized food grains to the poor.
- It prevents open-market price fluctuations by controlling the distribution of commodities. (Hence statement 1 is correct)
- The Indian PDS is a comprehensive network established in response to food shortages during World War II.
- It ensures accessible supplies to consumers across the country through fair price shops (FPS).
- Key commodities distributed: Wheat, Rice, Sugar, Kerosene, Edible oil, etc.
- Ministry of Consumer Affairs, Food and Public Distribution oversees the system.
- Department of Food and Public Distribution is responsible for overseeing the procurement and storage of food grains in the Ministry of Consumer Affairs, Food and Public Distribution. (Hence statement 2 is incorrect)
- Tamil Nadu stands out for effective PDS implementation.
Limitations of the Current PDS:
- Identification of eligible households remains a challenge.
- Discrepancies between food grain procurement and production levels.
- Insufficient storage facilities for food grains.
- High food subsidy burden on the government.
- Leakage and diversion of food grains intended for the poor.
- Lack of adequate infrastructure, including FPS and storage facilities.
Revamping the PDS:
- The introduction of Targeted Public Distribution System (TPDS) aimed to provide subsidised food to the poor.
- TPDS involves procuring food grains from farmers and delivering them to ration shops for beneficiaries.
- The National Food Security Act, 2013, made the right to food justiciable.
- Focus on identifying eligible households accurately.
- Emphasis on strengthening procurement and storage facilities.
- Addressing issues related to food subsidy management.
- Implementing measures to prevent leakage and diversion.
- Exploring alternatives to enhance TPDS efficiency.
Objectives of the Public Distribution System:
- Ensure availability of essential commodities at fair prices to all, especially vulnerable sections.
- Correct supply-demand imbalances and prevent hoarding and black marketing. (Hence statement 3 is correct)
- Distribute basic necessities equitably, promoting social justice.
- Stabilize prices and availability of mass consumption goods.
- Support poverty-alleviation programs, rural employment, and educational feeding initiatives.
Incorrect
Solution (b)
Public Distribution System (PDS): Overview, Functioning, Limitations, and Revamping
Overview and Functioning of PDS:
- PDS plays a vital role in ensuring food security by supplying subsidized food grains to the poor.
- It prevents open-market price fluctuations by controlling the distribution of commodities. (Hence statement 1 is correct)
- The Indian PDS is a comprehensive network established in response to food shortages during World War II.
- It ensures accessible supplies to consumers across the country through fair price shops (FPS).
- Key commodities distributed: Wheat, Rice, Sugar, Kerosene, Edible oil, etc.
- Ministry of Consumer Affairs, Food and Public Distribution oversees the system.
- Department of Food and Public Distribution is responsible for overseeing the procurement and storage of food grains in the Ministry of Consumer Affairs, Food and Public Distribution. (Hence statement 2 is incorrect)
- Tamil Nadu stands out for effective PDS implementation.
Limitations of the Current PDS:
- Identification of eligible households remains a challenge.
- Discrepancies between food grain procurement and production levels.
- Insufficient storage facilities for food grains.
- High food subsidy burden on the government.
- Leakage and diversion of food grains intended for the poor.
- Lack of adequate infrastructure, including FPS and storage facilities.
Revamping the PDS:
- The introduction of Targeted Public Distribution System (TPDS) aimed to provide subsidised food to the poor.
- TPDS involves procuring food grains from farmers and delivering them to ration shops for beneficiaries.
- The National Food Security Act, 2013, made the right to food justiciable.
- Focus on identifying eligible households accurately.
- Emphasis on strengthening procurement and storage facilities.
- Addressing issues related to food subsidy management.
- Implementing measures to prevent leakage and diversion.
- Exploring alternatives to enhance TPDS efficiency.
Objectives of the Public Distribution System:
- Ensure availability of essential commodities at fair prices to all, especially vulnerable sections.
- Correct supply-demand imbalances and prevent hoarding and black marketing. (Hence statement 3 is correct)
- Distribute basic necessities equitably, promoting social justice.
- Stabilize prices and availability of mass consumption goods.
- Support poverty-alleviation programs, rural employment, and educational feeding initiatives.
-
Question 16 of 35
16. Question
With reference to the Organic and Natural Farming, consider the following statements:
- India stands out with the remarkable achievement of hosting the largest number of organic farmers on a global scale, boasting a substantial population actively participating in organic farming endeavors.
- Himachal Pradesh willingly adopted organic farming and subsequently emerged as the world’s inaugural fully organic state.
- The Mission Organic Value Chain Development for North Eastern Region (MOVCDNER) promotes organic cultivation of specialized crops within the North East Region by leveraging Farmer Producer Organizations (FPOs).
How many of the above statements are correct?
Correct
Solution (b)
Organic and Natural Farming Benefits:
- Organic and natural farming offers chemical-free food grains and crops, enhancing soil health and reducing environmental pollution.
- India boasts 44.3 lakh organic farmers, the highest globally. (Hence statement 1 is correct)
- Approximately 59.1 lakh hectares were dedicated to organic farming by 2021-22.
Sikkim’s Pioneering Organic Endeavor:
- Sikkim voluntarily embraced organic farming, commencing the process in 2010.
- It became the world’s first fully organic state, inspiring similar goals in other regions like Tripura and Uttarakhand. (Hence statement 2 is incorrect)
Government Initiatives to Promote Organic Farming:
- The Government introduced two dedicated schemes:
- Paramparagat Krishi Vikas Yojana (PKVY)
- Mission Organic Value Chain Development for North Eastern Region (MOVCDNER)
- PKVY is implemented in clusters, with a focus on forming Farmer Producer Organisations (FPOs).
- Financial assistance, including incentives for organic inputs, is provided to farmers under PKVY.
- Namami Gange Programme covers a significant land area under organic farming.
- MOVCDNER encourages organic farming of niche crops in the North East Region through FPOs. (Hence statement 3 is correct)
Natural Farming Promotion:
- Bhartiya Prakratik Krishi Paddhati (BPKP) sub-scheme under PKVY launched in 2019-20.
- BPKP supports adoption of traditional indigenous practices, including Zero-Budget Natural Farming (ZBNF).
- Focus areas include capacity building, training, champion farmer demonstrations, and on-field guidance.
- BPKP has led to 4.09 lakh hectares of land adopting natural farming in eight states.
- Andhra Pradesh, Chhattisgarh, Kerala, Himachal Pradesh, Jharkhand, Odisha, Madhya Pradesh, and Tamil Nadu.
Incorrect
Solution (b)
Organic and Natural Farming Benefits:
- Organic and natural farming offers chemical-free food grains and crops, enhancing soil health and reducing environmental pollution.
- India boasts 44.3 lakh organic farmers, the highest globally. (Hence statement 1 is correct)
- Approximately 59.1 lakh hectares were dedicated to organic farming by 2021-22.
Sikkim’s Pioneering Organic Endeavor:
- Sikkim voluntarily embraced organic farming, commencing the process in 2010.
- It became the world’s first fully organic state, inspiring similar goals in other regions like Tripura and Uttarakhand. (Hence statement 2 is incorrect)
Government Initiatives to Promote Organic Farming:
- The Government introduced two dedicated schemes:
- Paramparagat Krishi Vikas Yojana (PKVY)
- Mission Organic Value Chain Development for North Eastern Region (MOVCDNER)
- PKVY is implemented in clusters, with a focus on forming Farmer Producer Organisations (FPOs).
- Financial assistance, including incentives for organic inputs, is provided to farmers under PKVY.
- Namami Gange Programme covers a significant land area under organic farming.
- MOVCDNER encourages organic farming of niche crops in the North East Region through FPOs. (Hence statement 3 is correct)
Natural Farming Promotion:
- Bhartiya Prakratik Krishi Paddhati (BPKP) sub-scheme under PKVY launched in 2019-20.
- BPKP supports adoption of traditional indigenous practices, including Zero-Budget Natural Farming (ZBNF).
- Focus areas include capacity building, training, champion farmer demonstrations, and on-field guidance.
- BPKP has led to 4.09 lakh hectares of land adopting natural farming in eight states.
- Andhra Pradesh, Chhattisgarh, Kerala, Himachal Pradesh, Jharkhand, Odisha, Madhya Pradesh, and Tamil Nadu.
-
Question 17 of 35
17. Question
Regarding the recently introduced ‘Krishi Megh,’ consider the following statements:
- It is the data recovery center positioned within the premises of the National Academy of Agricultural Research Management (NAARM) in Hyderabad.
- This endeavor has been initiated as part of the National Agricultural Higher Education Project (NAHEP), which receives funding from both the Indian government and the World Bank.
Choose the correct code:
Correct
Solution (c)
- In order to safeguard the invaluable data of the esteemed government research institution, the Indian Council of Agricultural Research (ICAR), the Agriculture Minister inaugurated a data recovery centre named ‘KRISHI MEGH’ on August 11, 2020. This centre, part of the National Agricultural Research & Education System – Cloud Infrastructure and Services, has been established in Hyderabad. The primary data centre of ICAR currently resides at the Indian Agricultural Statistics Research Institute (IASRI) in Delhi, which is situated in a seismic zone, posing a risk of data loss. Therefore, the establishment of a data recovery centre in a secure location is crucial to preserve our vital agricultural-related information.
- The data recovery centre, known as Krishi Megh, is situated at the National Academy of Agricultural Research Management (NAARM) in Hyderabad. Krishi Megh aims to integrate the ICAR-Data Centre at ICAR-IASRI in New Delhi with the Disaster Recovery Centre at ICAR-NAARM in Hyderabad. This initiative has been launched under the National Agricultural Higher Education Project (NAHEP), funded jointly by the Government of India and the World Bank. (Hence both the statements are correct)
The selection of NAARM, Hyderabad, as the location for Krishi Megh is strategic due to its distinct seismic zone when compared to the Data Centre at ICAR-IASRI in New Delhi. Moreover, Hyderabad offers a suitable environment with skilled IT professionals and favourable climatic conditions, including low humidity levels that can be controlled within the data centre environment.
Incorrect
Solution (c)
- In order to safeguard the invaluable data of the esteemed government research institution, the Indian Council of Agricultural Research (ICAR), the Agriculture Minister inaugurated a data recovery centre named ‘KRISHI MEGH’ on August 11, 2020. This centre, part of the National Agricultural Research & Education System – Cloud Infrastructure and Services, has been established in Hyderabad. The primary data centre of ICAR currently resides at the Indian Agricultural Statistics Research Institute (IASRI) in Delhi, which is situated in a seismic zone, posing a risk of data loss. Therefore, the establishment of a data recovery centre in a secure location is crucial to preserve our vital agricultural-related information.
- The data recovery centre, known as Krishi Megh, is situated at the National Academy of Agricultural Research Management (NAARM) in Hyderabad. Krishi Megh aims to integrate the ICAR-Data Centre at ICAR-IASRI in New Delhi with the Disaster Recovery Centre at ICAR-NAARM in Hyderabad. This initiative has been launched under the National Agricultural Higher Education Project (NAHEP), funded jointly by the Government of India and the World Bank. (Hence both the statements are correct)
The selection of NAARM, Hyderabad, as the location for Krishi Megh is strategic due to its distinct seismic zone when compared to the Data Centre at ICAR-IASRI in New Delhi. Moreover, Hyderabad offers a suitable environment with skilled IT professionals and favourable climatic conditions, including low humidity levels that can be controlled within the data centre environment.
-
Question 18 of 35
18. Question
Consider the following statements with respect to the Agriculture Infrastructure Fund (AIF) which is operational from 2020-21 to 2032-33:
- It provides backing for the development of infrastructure related to pre-harvest management and community farming assets.
- It provides advantages such as a 3% interest subvention and support through credit guarantees.
- The AIF scheme has the potential to partner with other State or Central Government initiatives, demonstrating the possibility of substantial investments in the agriculture sector.
How many of the above statements are correct?
Correct
Solution (b)
Agriculture Infrastructure Fund (AIF):
- The AIF, operational from 2020-21 to 2032-33, supports the establishment of post-harvest management infrastructure and community farming assets. (Hence statement 1is incorrect)
- It offers benefits like 3% interest subvention and credit guarantee assistance. (Hence statement 2 is correct)
- A provision of ₹1 lakh crore for 2020-21 to 2025-26 has been allocated, with interest subvention and credit guarantee extending until 2032-33.
- The AIF scheme can collaborate with other State or Central Government programs, showcasing potential for significant investments in agriculture. (Hence statement 3 is correct)
Incorrect
Solution (b)
Agriculture Infrastructure Fund (AIF):
- The AIF, operational from 2020-21 to 2032-33, supports the establishment of post-harvest management infrastructure and community farming assets. (Hence statement 1is incorrect)
- It offers benefits like 3% interest subvention and credit guarantee assistance. (Hence statement 2 is correct)
- A provision of ₹1 lakh crore for 2020-21 to 2025-26 has been allocated, with interest subvention and credit guarantee extending until 2032-33.
- The AIF scheme can collaborate with other State or Central Government programs, showcasing potential for significant investments in agriculture. (Hence statement 3 is correct)
-
Question 19 of 35
19. Question
With reference to the Unified Farmer Service Platform (UFSP), consider the following statements:
- UFSP acts as a cohesive system enabling seamless interoperability among public and private IT systems in the national agricultural ecosystem.
- UFSP allows registration for both Service Providers and services meant for farmers.
- UFSP enables the exchange of data between different schemes and services, guaranteeing inclusive service delivery to farmers.
How many of the above statements are correct?
Correct
Solution (c)
Unified Farmer Service Platform (UFSP)
Unified Farmer Service Platform (UFSP) serves as a unified system with Core Infrastructure, Data, Applications, and Tools, facilitating seamless interoperability among public and private IT systems within the agriculture ecosystem across the country. (Hence statement 1 is correct)
The roles of UFSP include:
- Central Agency Role: Comparable to UPI in e-Payments, UFSP acts as a central agency within the agricultural ecosystem.
- Registration Facilitation: UFSP enables registration for both Service Providers (public and private) and Farmer Services. (Hence statement 2 is correct)
- Enforcement of Service Rules: UFSP enforces essential rules and validations during the service delivery process.
- Standard Repository: It serves as a repository for all relevant standards, APIs (Application Programming Interface), and formats.
- Data Exchange Medium: UFSP facilitates data exchange among various schemes and services, ensuring comprehensive service delivery to farmers. (Hence statement 3 is correct)
Incorrect
Solution (c)
Unified Farmer Service Platform (UFSP)
Unified Farmer Service Platform (UFSP) serves as a unified system with Core Infrastructure, Data, Applications, and Tools, facilitating seamless interoperability among public and private IT systems within the agriculture ecosystem across the country. (Hence statement 1 is correct)
The roles of UFSP include:
- Central Agency Role: Comparable to UPI in e-Payments, UFSP acts as a central agency within the agricultural ecosystem.
- Registration Facilitation: UFSP enables registration for both Service Providers (public and private) and Farmer Services. (Hence statement 2 is correct)
- Enforcement of Service Rules: UFSP enforces essential rules and validations during the service delivery process.
- Standard Repository: It serves as a repository for all relevant standards, APIs (Application Programming Interface), and formats.
- Data Exchange Medium: UFSP facilitates data exchange among various schemes and services, ensuring comprehensive service delivery to farmers. (Hence statement 3 is correct)
-
Question 20 of 35
20. Question
With reference to the Micro Irrigation Fund (MIF) for Irrigation, consider the following statements:
- Cabinet Committee on Economic Affairs (CCEA) approved an initial corpus of Rs. 5,000 crores for a dedicated MIF under Pradhan Mantri Fasal Bhima Yojana (PMFBY) in NABARD.
- The goal of MIF is to irrigate approximately one million hectares of land through innovative micro irrigation projects.
- Micro irrigation coverage expands through community-driven and innovative projects initiated by the state.
How many of the above statements are correct?
Correct
Solution (b)
Micro Irrigation Fund (MIF) for Irrigation:
Initiative Approval:
- Cabinet Committee on Economic Affairs (CCEA) approved an initial corpus of Rs. 5,000 crores for a dedicated MIF under Pradhan Mantri Krishi Sinchayee Yojana (PMKSY) in NABARD. (Hence statement 1 is incorrect)
Allocation and Loan Extension:
- MIF allocation: Rs. 2,000 crores in 2018-19 and Rs. 3,000 crores in 2019-20.
- NABARD extends loans to state governments under MIF.
- Proposed lending rate: 3% lower than NABARD’s cost of raising funds.
Loan Repayment:
- States repay loans over a total of seven years, including a two-year grace period.
- Estimated financial implication of interest subvention: about Rs. 750 crores.
Benefits:
- MIF complements Per Drop More Crop (PDMC) Component of PMKSY effectively.
- Aims to bring around 10 lakh ha of land under irrigation using innovative micro irrigation projects. (Hence statement 2 is correct)
- Facilitates states in resource mobilization, additional subsidy implementation, and achieving annual targets.
Implementation Strategy:
- States access MIF for integrated irrigation projects, including PPP mode.
- Supports micro irrigation through top-up subsidies.
- Available for Farmers Producers Organizations (FPOs), Cooperatives, State Level Agencies with state guarantee or collateral.
- Funds accessible for innovative cluster-based Community Irrigation Projects.
Advisory and Steering Committees:
- Advisory Committee provides policy direction, coordination, and project monitoring.
- Steering Committee examines and approves state projects.
Coverage:
- Micro Irrigation Fund (MIF) covers the entire nation.
- Encourages states lagging in micro irrigation adoption to benefit from the fund.
- State-driven community and innovative projects expand micro irrigation coverage. (Hence statement 3 is correct)
Incorrect
Solution (b)
Micro Irrigation Fund (MIF) for Irrigation:
Initiative Approval:
- Cabinet Committee on Economic Affairs (CCEA) approved an initial corpus of Rs. 5,000 crores for a dedicated MIF under Pradhan Mantri Krishi Sinchayee Yojana (PMKSY) in NABARD. (Hence statement 1 is incorrect)
Allocation and Loan Extension:
- MIF allocation: Rs. 2,000 crores in 2018-19 and Rs. 3,000 crores in 2019-20.
- NABARD extends loans to state governments under MIF.
- Proposed lending rate: 3% lower than NABARD’s cost of raising funds.
Loan Repayment:
- States repay loans over a total of seven years, including a two-year grace period.
- Estimated financial implication of interest subvention: about Rs. 750 crores.
Benefits:
- MIF complements Per Drop More Crop (PDMC) Component of PMKSY effectively.
- Aims to bring around 10 lakh ha of land under irrigation using innovative micro irrigation projects. (Hence statement 2 is correct)
- Facilitates states in resource mobilization, additional subsidy implementation, and achieving annual targets.
Implementation Strategy:
- States access MIF for integrated irrigation projects, including PPP mode.
- Supports micro irrigation through top-up subsidies.
- Available for Farmers Producers Organizations (FPOs), Cooperatives, State Level Agencies with state guarantee or collateral.
- Funds accessible for innovative cluster-based Community Irrigation Projects.
Advisory and Steering Committees:
- Advisory Committee provides policy direction, coordination, and project monitoring.
- Steering Committee examines and approves state projects.
Coverage:
- Micro Irrigation Fund (MIF) covers the entire nation.
- Encourages states lagging in micro irrigation adoption to benefit from the fund.
- State-driven community and innovative projects expand micro irrigation coverage. (Hence statement 3 is correct)
-
Question 21 of 35
21. Question
Consider the following statements about the Members of Parliament Local Area Development Scheme
- It is a central sector scheme that enables the Members of Parliament to recommend works for the creation of durable community assets based on locally felt needs.
- Under the scheme, the annual MPLADS fund entitlement per MP constituency is Rs. 10 crore.
- While the Lok Sabha members can recommend works within their constituencies and the Rajya Sabha members can recommend works in one or more districts in the State from where he/she has been elected.
- The nominated members of the Lok Sabha and Rajya Sabha may select any one or more Districts from any one state in the country for implementation of their choice of work under the scheme.
How many of the above statements are correct?
Correct
Solution (c)
- The Members of Parliament Local Area Development Scheme is a central sector scheme that enables the Members of Parliament to recommend works for the creation of durable community assets based on locally felt needs. Hence statement 1 is correct.
- The role of an MP is, however, limited to the recommendation of a project.
- The onus is on the district authority to sanction, execute, and complete the recommended project within a particular timeframe.
- Under the scheme, the annual MPLADS fund entitlement per MP constituency is Rs. 5 crore. Hence statement 2 is incorrect.
- The Ministry of Statistics and Programme Implementation has been responsible for the policy formulation, release of funds, and prescribing monitoring mechanisms for the implementation of the scheme.
- While the Lok Sabha members can recommend works within their constituencies and the Rajya Sabha members can recommend works in one or more districts in the State from where he/she has been elected. Hence statement 3 is correct.
- MPs are to recommend every year, works costing at least 15 percent of the MPLADS entitlement for the year for areas inhabited by the Scheduled Caste population and 7.5 percent for areas inhabited by the S.T. population.
- In case an elected MP wishes to contribute MPLADS funds outside the constituency or the State/UT, they can recommend working up to Rs 25 lakh in a financial year.
- MPs do not directly receive funds under MPLADS. The Centre directly transfers the sanctioned amount in two installments of Rs. 2.5 crore to the district authorities of the concerned MP’s nodal district after a recommended project gets approval.
- The nominated members of the Lok Sabha and Rajya Sabha may select any one or more Districts from any one state in the country for implementation of their choice of work under the scheme. Hence statement 4 is correct.
Incorrect
Solution (c)
- The Members of Parliament Local Area Development Scheme is a central sector scheme that enables the Members of Parliament to recommend works for the creation of durable community assets based on locally felt needs. Hence statement 1 is correct.
- The role of an MP is, however, limited to the recommendation of a project.
- The onus is on the district authority to sanction, execute, and complete the recommended project within a particular timeframe.
- Under the scheme, the annual MPLADS fund entitlement per MP constituency is Rs. 5 crore. Hence statement 2 is incorrect.
- The Ministry of Statistics and Programme Implementation has been responsible for the policy formulation, release of funds, and prescribing monitoring mechanisms for the implementation of the scheme.
- While the Lok Sabha members can recommend works within their constituencies and the Rajya Sabha members can recommend works in one or more districts in the State from where he/she has been elected. Hence statement 3 is correct.
- MPs are to recommend every year, works costing at least 15 percent of the MPLADS entitlement for the year for areas inhabited by the Scheduled Caste population and 7.5 percent for areas inhabited by the S.T. population.
- In case an elected MP wishes to contribute MPLADS funds outside the constituency or the State/UT, they can recommend working up to Rs 25 lakh in a financial year.
- MPs do not directly receive funds under MPLADS. The Centre directly transfers the sanctioned amount in two installments of Rs. 2.5 crore to the district authorities of the concerned MP’s nodal district after a recommended project gets approval.
- The nominated members of the Lok Sabha and Rajya Sabha may select any one or more Districts from any one state in the country for implementation of their choice of work under the scheme. Hence statement 4 is correct.
-
Question 22 of 35
22. Question
Consider the following statements about Vakalatnama
- It is a legal document in India that authorizes an advocate to represent a party in court proceedings.
- It is defined in the Civil Procedure Code of 1908 and the Power of Attorney Act of 1882.
- It outlines the scope of the lawyer’s authority, the specific case or matter for which representation is sought, and other relevant details.
How many of the above statements are correct?
Correct
Solution (b)
- Vakalatnama is a legal document in India that authorizes an advocate to represent a party in court proceedings. Hence statement 1 is correct.
- The term “Vakalatnama” is derived from two words: “Vakalat,” which means authority or power of attorney, and “Nama,” which means a document.
- When a person engages the services of a lawyer to represent them in a legal matter, they sign a Vakalatnama, which formally appoints the lawyer to act on their behalf.
- This document outlines the scope of the lawyer’s authority, the specific case or matter for which representation is sought, and other relevant details.
- It is also known as a memo of appearance, Vakilat Patra, VP.
- It is not defined in the Civil Procedure Code of 1908 and the Power of Attorney Act of 1882. Hence statement 2 is incorrect.
- The meaning of Vakalatnama is defined in the Advocates Welfare Fund Act, 2001, under which “Vakalatnama” includes a memorandum of appearance or any other document by which an advocate is empowered to appear or plead before any court, tribunal, or other authority.
- The holder of the vakalatnama is called a pleader, an advocate, counsel, vakil, or an attorney who is authorized to accept the vakalatnama on behalf of his client or party of the litigation.
- It outlines the scope of the lawyer’s authority, the specific case or matter for which representation is sought, and other relevant details. Hence statement 3 is correct.
- Vakalatnama can be authorized by the following:
- An aggrieved person can authorize a Vakalatnama.
- Anybody holding the Power of Attorney for the aggrieved person.
- Anybody representing the aggrieved person in business or trade in that jurisdiction.
- A Vakalatnama can also be authorized by a joint party in a case in order to appoint either a set of advocates or the same advocate.
Incorrect
Solution (b)
- Vakalatnama is a legal document in India that authorizes an advocate to represent a party in court proceedings. Hence statement 1 is correct.
- The term “Vakalatnama” is derived from two words: “Vakalat,” which means authority or power of attorney, and “Nama,” which means a document.
- When a person engages the services of a lawyer to represent them in a legal matter, they sign a Vakalatnama, which formally appoints the lawyer to act on their behalf.
- This document outlines the scope of the lawyer’s authority, the specific case or matter for which representation is sought, and other relevant details.
- It is also known as a memo of appearance, Vakilat Patra, VP.
- It is not defined in the Civil Procedure Code of 1908 and the Power of Attorney Act of 1882. Hence statement 2 is incorrect.
- The meaning of Vakalatnama is defined in the Advocates Welfare Fund Act, 2001, under which “Vakalatnama” includes a memorandum of appearance or any other document by which an advocate is empowered to appear or plead before any court, tribunal, or other authority.
- The holder of the vakalatnama is called a pleader, an advocate, counsel, vakil, or an attorney who is authorized to accept the vakalatnama on behalf of his client or party of the litigation.
- It outlines the scope of the lawyer’s authority, the specific case or matter for which representation is sought, and other relevant details. Hence statement 3 is correct.
- Vakalatnama can be authorized by the following:
- An aggrieved person can authorize a Vakalatnama.
- Anybody holding the Power of Attorney for the aggrieved person.
- Anybody representing the aggrieved person in business or trade in that jurisdiction.
- A Vakalatnama can also be authorized by a joint party in a case in order to appoint either a set of advocates or the same advocate.
-
Question 23 of 35
23. Question
Consider the following statements about Whooping Cough
- It is a highly contagious viral infection which impacts the respiratory tract.
- It spreads via droplets when an infected person coughs or sneezes.
- It can be treated with Diphtheria, tetanus, and pertussis (DTaP) vaccines.
How many of the above statements are correct?
Correct
Solution (b)
- Whooping Cough is a highly contagious bacterial infection that impacts the respiratory tract. Hence statement 1 is incorrect.
- It is particularly severe in infants and in case of complications may lead to pneumonia and even death.
- It spreads via droplets when an infected person coughs or sneezes. Hence statement 2 is correct.
- It attaches itself to the throat lining and then produces toxins that damage hair-like projections and clear debris and mucus.
- This leads to inflammation in the airways and causes severe coughing fits. It also then leads to difficulty in breathing.
- It begins with cold-like symptoms accompanied by low-grade fever. As it progresses, the cough gets accompanied by a “whooping” sound and these spells may last for a while leading to vomiting.
- It can be treated with Diphtheria, tetanus, and pertussis (DTaP) vaccines. Hence statement 3 is correct.
Incorrect
Solution (b)
- Whooping Cough is a highly contagious bacterial infection that impacts the respiratory tract. Hence statement 1 is incorrect.
- It is particularly severe in infants and in case of complications may lead to pneumonia and even death.
- It spreads via droplets when an infected person coughs or sneezes. Hence statement 2 is correct.
- It attaches itself to the throat lining and then produces toxins that damage hair-like projections and clear debris and mucus.
- This leads to inflammation in the airways and causes severe coughing fits. It also then leads to difficulty in breathing.
- It begins with cold-like symptoms accompanied by low-grade fever. As it progresses, the cough gets accompanied by a “whooping” sound and these spells may last for a while leading to vomiting.
- It can be treated with Diphtheria, tetanus, and pertussis (DTaP) vaccines. Hence statement 3 is correct.
-
Question 24 of 35
24. Question
Consider the following statements about Methanol
- It is produced based on the direct combination of carbon monoxide gas and hydrogen in the presence of a catalyst.
- It is used to make chemicals, to remove water from automotive and aviation fuels, and as a solvent for paints and plastics.
- It has a higher risk of flammability compared to gasoline.
How many of the above statements are correct?
Correct
Solution (b)
- Methanol is produced based on the direct combination of carbon monoxide gas and hydrogen in the presence of a catalyst. Hence statement 1 is correct.
- It can be manufactured from a variety of domestic carbon-based feedstocks, such as biomass, natural gas, and coal.
- It appears as a colourless fairly volatile liquid with a faintly sweet pungent odour like that of ethyl alcohol.
- It is also known as wood alcohol. It can completely mix with water.
- It is used to make chemicals, to remove water from automotive and aviation fuels, and as a solvent for paints and plastics. Hence statement 2 is correct.
- Methanol is cheap to produce relative to other alternative fuels.
- It has a lower risk of flammability compared to gasoline. Hence statement 3 is incorrect.
Incorrect
Solution (b)
- Methanol is produced based on the direct combination of carbon monoxide gas and hydrogen in the presence of a catalyst. Hence statement 1 is correct.
- It can be manufactured from a variety of domestic carbon-based feedstocks, such as biomass, natural gas, and coal.
- It appears as a colourless fairly volatile liquid with a faintly sweet pungent odour like that of ethyl alcohol.
- It is also known as wood alcohol. It can completely mix with water.
- It is used to make chemicals, to remove water from automotive and aviation fuels, and as a solvent for paints and plastics. Hence statement 2 is correct.
- Methanol is cheap to produce relative to other alternative fuels.
- It has a lower risk of flammability compared to gasoline. Hence statement 3 is incorrect.
-
Question 25 of 35
25. Question
Consider the following statements about C-Dome
- It is a naval version of the Iron Dome air defence system.
- It is used to shield against rocket and missile attacks.
- Iron Dome is Germany’s air missile defence system that can defend against short-range rockets.
How many of the above statements are correct?
Correct
Solution (b)
- C-Dome is a naval version of the Iron Dome air defence system. Hence statement 1 is correct.
- The C-Dome, which was first unveiled in 2014, and declared operational inNovember 2022, works similarly to the Iron Dome, using some of the same technology, except that it’s mounted on ships.
- It is mounted on Sa’ar 6-class corvettes, German-made warships, and uses the same interceptor as the Iron Dome.
- Unlike the Iron Dome, which has its dedicated radar, the C-Dome is integrated into the ship’s radarto detect incoming targets.
- It is used to shield against rocket and missile attacks. Hence statement 2 is correct.
- C-Dome ensures full-circular vessel protectionand high kill probability against a full spectrum of modern threats—maritime and coastal.
- Iron Dome is Israel’s air missile defence system that can defend against short-range rockets. Hence statement 3 is incorrect.
- It is capable of successfully handling multiple rocketsat a time.
- It was developed by Rafael Advanced Defence Systemsand Israel Aerospace Industries, the system became operational in March 2011.
Incorrect
Solution (b)
- C-Dome is a naval version of the Iron Dome air defence system. Hence statement 1 is correct.
- The C-Dome, which was first unveiled in 2014, and declared operational inNovember 2022, works similarly to the Iron Dome, using some of the same technology, except that it’s mounted on ships.
- It is mounted on Sa’ar 6-class corvettes, German-made warships, and uses the same interceptor as the Iron Dome.
- Unlike the Iron Dome, which has its dedicated radar, the C-Dome is integrated into the ship’s radarto detect incoming targets.
- It is used to shield against rocket and missile attacks. Hence statement 2 is correct.
- C-Dome ensures full-circular vessel protectionand high kill probability against a full spectrum of modern threats—maritime and coastal.
- Iron Dome is Israel’s air missile defence system that can defend against short-range rockets. Hence statement 3 is incorrect.
- It is capable of successfully handling multiple rocketsat a time.
- It was developed by Rafael Advanced Defence Systemsand Israel Aerospace Industries, the system became operational in March 2011.
-
Question 26 of 35
26. Question
Consider the following pairs
Volcano Country 1. Tungurahua Guatemala 2. Pacaya Ecuador 3. Sakurajima Japan 4. Whakaari New Zealand How many given pairs are correctly matched?
Correct
Solution (b)
Volcano Country 1. Tungurahua Ecuador 2. Pacaya Guatemala 3. Sakurajima Japan 4. Whakaari New Zealand Hence option b is correct.
Volcanoes in the news: Redoubt in Alaska, Eyjafjallajökull, and Hekla in Iceland, Stromboli in Italy, Aso in Japan, Yasur in Vanuatu, and Momotombo in Nicaragua.
Incorrect
Solution (b)
Volcano Country 1. Tungurahua Ecuador 2. Pacaya Guatemala 3. Sakurajima Japan 4. Whakaari New Zealand Hence option b is correct.
Volcanoes in the news: Redoubt in Alaska, Eyjafjallajökull, and Hekla in Iceland, Stromboli in Italy, Aso in Japan, Yasur in Vanuatu, and Momotombo in Nicaragua.
-
Question 27 of 35
27. Question
Consider the following statements about Kala Azar
- It is also known as black fever or Dum-Dum fever, referring to the greyish or blackish discoloration ofthe skin during infection.
- It is caused by a bacteriaof the genus Leishmania, which is transmitted by the bite of infected female phlebotomine sandflies.
- It is treated through liposomal AmB, paromomycin, miltefosine, and multidrug therapy.
How many of the above statements are correct?
Correct
Solution (b)
- Kala Azar, or visceral leishmaniasis, is the second deadliest parasitic diseasein the world after Malaria.
- It is also known as black fever or Dum-Dum fever, referring to the greyish or blackish discoloration of the skin during infection. Hence statement 1 is correct.
- It is one of the most dangerous neglected tropical diseases (NTDs) and is endemic in 76 countries.
- It is caused by a protozoan parasiteof the genus Leishmania, which is transmitted by the bite of infected female phlebotomine sandflies. Hence statement 2 is incorrect.
- The parasite primarily infects the reticuloendothelial systemand may be found in abundance in the bone marrow, spleen, and liver.
- Post Kala-azar Dermal Leishmaniasis (PKDL)is a condition when Leishmania
donovani invades skin cells, resides and develops there, and manifests as dermal leisions. Some of the kala-azar cases manifest PKD after a few years of treatment.
- The disease affects some of the poorest peopleand is linked to malnutrition, population displacement, poor housing, a weak immune system, and a lack of financial resources.
- Kala-azar is characterized by irregular bouts of fever, substantial weight loss, swelling of the spleenand liver, and severe anaemia.
- If the disease is not treatedearly and in time, affected individuals can die within two years.
- It is treated through liposomal AmB, paromomycin, miltefosine, and multidrug therapy. Hence statement 3 is correct.
Incorrect
Solution (b)
- Kala Azar, or visceral leishmaniasis, is the second deadliest parasitic diseasein the world after Malaria.
- It is also known as black fever or Dum-Dum fever, referring to the greyish or blackish discoloration of the skin during infection. Hence statement 1 is correct.
- It is one of the most dangerous neglected tropical diseases (NTDs) and is endemic in 76 countries.
- It is caused by a protozoan parasiteof the genus Leishmania, which is transmitted by the bite of infected female phlebotomine sandflies. Hence statement 2 is incorrect.
- The parasite primarily infects the reticuloendothelial systemand may be found in abundance in the bone marrow, spleen, and liver.
- Post Kala-azar Dermal Leishmaniasis (PKDL)is a condition when Leishmania
donovani invades skin cells, resides and develops there, and manifests as dermal leisions. Some of the kala-azar cases manifest PKD after a few years of treatment.
- The disease affects some of the poorest peopleand is linked to malnutrition, population displacement, poor housing, a weak immune system, and a lack of financial resources.
- Kala-azar is characterized by irregular bouts of fever, substantial weight loss, swelling of the spleenand liver, and severe anaemia.
- If the disease is not treatedearly and in time, affected individuals can die within two years.
- It is treated through liposomal AmB, paromomycin, miltefosine, and multidrug therapy. Hence statement 3 is correct.
-
Question 28 of 35
28. Question
Consider the following statements about the International Narcotics Control Board
- It is the independent and quasi-judicial monitoringbody for the implementation of the United Nations international drug control conventions.
- It monitors governments’ control over chemicals used in the illicit manufacture of drugs and assists them in preventing the diversion of those chemicals into illicit traffic.
Choose the correct code:
Correct
Solution (c)
- The International Narcotics Control Board is the independent and quasi-judicial monitoring body for the implementation of the United Nations international drug control conventions. Hence statement 1 is correct.
- It was established by the Single Conventionon Narcotic Drugs of 1961 by merging two bodies: the Permanent Central Narcotics Board, created by the 1925 International Opium Convention; and the Drug Supervisory Body, created by the 1931 Convention for Limiting the Manufacture and Regulating the Distribution of Narcotic Drugs.
- It consists of 13 members who are elected by the Economic and Social Council and who serve in their capacity, not as government representatives.
- Three memberswith medical, pharmacological, or pharmaceutical experience are elected from a list of persons nominated by the World Health Organization (WHO) and 10 members are elected from a list of persons nominated by Governments.
- It monitors governments’ control over chemicals used in the illicit manufacture of drugs and assists them in preventing the diversion of those chemicals into illicit traffic. Hence statement 2 is correct.
- It endeavours in cooperation with governments, to ensure that adequate supplies of drugs are available for medical and scientific usesand that the diversion of drugs from licit sources to illicit channels does not occur.
Incorrect
Solution (c)
- The International Narcotics Control Board is the independent and quasi-judicial monitoring body for the implementation of the United Nations international drug control conventions. Hence statement 1 is correct.
- It was established by the Single Conventionon Narcotic Drugs of 1961 by merging two bodies: the Permanent Central Narcotics Board, created by the 1925 International Opium Convention; and the Drug Supervisory Body, created by the 1931 Convention for Limiting the Manufacture and Regulating the Distribution of Narcotic Drugs.
- It consists of 13 members who are elected by the Economic and Social Council and who serve in their capacity, not as government representatives.
- Three memberswith medical, pharmacological, or pharmaceutical experience are elected from a list of persons nominated by the World Health Organization (WHO) and 10 members are elected from a list of persons nominated by Governments.
- It monitors governments’ control over chemicals used in the illicit manufacture of drugs and assists them in preventing the diversion of those chemicals into illicit traffic. Hence statement 2 is correct.
- It endeavours in cooperation with governments, to ensure that adequate supplies of drugs are available for medical and scientific usesand that the diversion of drugs from licit sources to illicit channels does not occur.
-
Question 29 of 35
29. Question
Consider the following statements about Sungrazing Comets
- They are a special class of comets that come very close to the sun at their nearest approach, a point called perihelion.
- They follow a similar orbit, called the Kreutz Path, a single orbit that takes 800 years to complete.
Choose the correct code:
Correct
Solution (c)
- Sungrazing Comets are a special class of comets that come very close to the sun at their nearest approach, a point called perihelion. Hence statement 1 is correct.
- To be considered a sungrazer, a comet needs to get within about 850,000 miles from the sun at perihelion.
- Many come even closer, even to within a few thousand miles.
- Being so close to the sun is very hard on comets for many reasons. They are:
- They are subjected to a lot of solar radiation, which boils off their water or other volatiles.
- The physical push of the radiation and the solar wind also help form the tails.
- As they get closer to the sun, the comets experience extremely strong tidal forces or gravitational stress.
- In this hostile environment, many sungrazers do not survive their trip around the sun.
- Although they don’t actually crash into the solar surface, the sun is able to destroy them.
- Most usually evaporate in the hot solar atmosphere.
- They follow a similar orbit, called the Kreutz Path, a single orbit that takes 800 years to complete. Hence statement 2 is correct.
- They collectively belong to a population called the Kreutz Group.
- These Kreutz comets are fragments from a single large comet that was shattered thousands of years ago.
- The far end of the Kreutz path lies 160 times farther from the sun than the orbit of Earth.
Incorrect
Solution (c)
- Sungrazing Comets are a special class of comets that come very close to the sun at their nearest approach, a point called perihelion. Hence statement 1 is correct.
- To be considered a sungrazer, a comet needs to get within about 850,000 miles from the sun at perihelion.
- Many come even closer, even to within a few thousand miles.
- Being so close to the sun is very hard on comets for many reasons. They are:
- They are subjected to a lot of solar radiation, which boils off their water or other volatiles.
- The physical push of the radiation and the solar wind also help form the tails.
- As they get closer to the sun, the comets experience extremely strong tidal forces or gravitational stress.
- In this hostile environment, many sungrazers do not survive their trip around the sun.
- Although they don’t actually crash into the solar surface, the sun is able to destroy them.
- Most usually evaporate in the hot solar atmosphere.
- They follow a similar orbit, called the Kreutz Path, a single orbit that takes 800 years to complete. Hence statement 2 is correct.
- They collectively belong to a population called the Kreutz Group.
- These Kreutz comets are fragments from a single large comet that was shattered thousands of years ago.
- The far end of the Kreutz path lies 160 times farther from the sun than the orbit of Earth.
-
Question 30 of 35
30. Question
Consider the following statements about the Jenu Kuruba Community
- They are a traditional honey-gathering tribe inhabiting Karnataka, Kerala, and Tamil Nadu.
- They practice shifting cultivation and live in small settlements called
Choose the correct code:
Correct
Solution (c)
- The Jenu Kuruba Community is a traditional honey-gathering tribe inhabiting Karnataka, Kerala, and Tamil Nadu. Hence statement 1 is correct.
- The main occupation used to be food gathering in the forests, collection of minor forest produce in the forests, and collection of minor forest produce including honey.
- They practice shifting cultivation and live in small settlements called Hadi. Hence statement 2 is correct.
- People of this community live a semi-nomadic lifestyle which is not maintained by rulers, police, centralized visible forces, or religious monasteries; but by the technique of its own discipline and diffused power.
- The pattern is that it is maintained at each settlement level with a head man(yajamana) and a ritual head/shaman (gudda).
Incorrect
Solution (c)
- The Jenu Kuruba Community is a traditional honey-gathering tribe inhabiting Karnataka, Kerala, and Tamil Nadu. Hence statement 1 is correct.
- The main occupation used to be food gathering in the forests, collection of minor forest produce in the forests, and collection of minor forest produce including honey.
- They practice shifting cultivation and live in small settlements called Hadi. Hence statement 2 is correct.
- People of this community live a semi-nomadic lifestyle which is not maintained by rulers, police, centralized visible forces, or religious monasteries; but by the technique of its own discipline and diffused power.
- The pattern is that it is maintained at each settlement level with a head man(yajamana) and a ritual head/shaman (gudda).
-
Question 31 of 35
31. Question
Manoj’s clock is such that it loses 4 minutes every hour. He set the clock right at 7:30 am on Sunday and kept it away. Few days later, he checked his clock and it was still showing the right time. When did Manoj check the clock again?
Correct
Solution (a)
To show the same time i.e. 7:30, the clock will have to lose 12 hours.
This is because 7:30 a.m. or 7:30 p.m. will not matter in this case.
The clock loses 4 minutes in 60 minutes (per hour).
Hence, time required to lose 720 minutes = (720/4) = 180 hours i.e. 7 days and 12 hours.
Hence, Manoj would have checked his clock at 7:30 p.m. on Sunday or 7:30 a.m. on Monday. Only the first value (7:30 p.m. on Sunday) is given in the options.
Hence, option a.
Incorrect
Solution (a)
To show the same time i.e. 7:30, the clock will have to lose 12 hours.
This is because 7:30 a.m. or 7:30 p.m. will not matter in this case.
The clock loses 4 minutes in 60 minutes (per hour).
Hence, time required to lose 720 minutes = (720/4) = 180 hours i.e. 7 days and 12 hours.
Hence, Manoj would have checked his clock at 7:30 p.m. on Sunday or 7:30 a.m. on Monday. Only the first value (7:30 p.m. on Sunday) is given in the options.
Hence, option a.
-
Question 32 of 35
32. Question
Akshay delivers a 20 litre solution of milk and water in the ratio 3 : 2 to Rakesh. Rakesh removed 10 litres from this, replaced it with pure milk and delivered it to Shashank. Shashank repeated this process and delivered it to Sana. What is the ratio of milk and water in Sana’s solution?
Correct
Solution (b)
Original quantity of milk in solution = (3/5) × 20 = 12 litres and original quantity of water = 8 litres.
Now, successive replacement happens twice (Rakesh to Shashank and Shashank to Sana).
Let the quantity of water in the final solution be w litres.
∴ w/8 = [1 – (10/20)]2
∴ w = 8 × (0.5)2 = 2 litres
∴ Quantity of water = 2 litres and quantity of milk = 18 litres
∴ Required ratio = 18 : 2 = 9 : 1
Hence, option b.
Incorrect
Solution (b)
Original quantity of milk in solution = (3/5) × 20 = 12 litres and original quantity of water = 8 litres.
Now, successive replacement happens twice (Rakesh to Shashank and Shashank to Sana).
Let the quantity of water in the final solution be w litres.
∴ w/8 = [1 – (10/20)]2
∴ w = 8 × (0.5)2 = 2 litres
∴ Quantity of water = 2 litres and quantity of milk = 18 litres
∴ Required ratio = 18 : 2 = 9 : 1
Hence, option b.
-
Question 33 of 35
33. Question
What is the angle subtended by the Basketball sector at the centre?
Correct
Solution (b)
Total degrees in a circle = 360 ̊
Percentage that basketball occupies in the pie chart = 20%
Therefore, angle subtended by basketball = (20/100) * 360 ̊ = 72 ̊
Incorrect
Solution (b)
Total degrees in a circle = 360 ̊
Percentage that basketball occupies in the pie chart = 20%
Therefore, angle subtended by basketball = (20/100) * 360 ̊ = 72 ̊
-
Question 34 of 35
34. Question
How many integers greater than 200 and less than 400 can be formed using only odd digits if the repetition of digits is allowed?
Correct
Solution (b)
The number to be formed is in the range 201-399 using the digits 1, 3, 5, 7, 9.
Using these digits, the hundreds digit can only be 3.
This can be chosen only in 1 way.
Since repetition is allowed, the other digits can be chosen in 5 ways each.
∴ Quantity of such numbers possible = 1 × 5 × 5 = 25
Hence, option b.
Incorrect
Solution (b)
The number to be formed is in the range 201-399 using the digits 1, 3, 5, 7, 9.
Using these digits, the hundreds digit can only be 3.
This can be chosen only in 1 way.
Since repetition is allowed, the other digits can be chosen in 5 ways each.
∴ Quantity of such numbers possible = 1 × 5 × 5 = 25
Hence, option b.
-
Question 35 of 35
35. Question
What is X in the sequence? 47 58 71 79 95 X
Correct
Solution (d)
47 + (4 + 7) = 58
58 + (5 + 8) = 71
71 + (7 + 1) = 79
79 + (7 + 9) = 95
95 + (9 + 5) = 109
Incorrect
Solution (d)
47 + (4 + 7) = 58
58 + (5 + 8) = 71
71 + (7 + 1) = 79
79 + (7 + 9) = 95
95 + (9 + 5) = 109
All the Best
IASbaba